RN Comprehensive Online Practice 2023 B

Pataasin ang iyong marka sa homework at exams ngayon gamit ang Quizwiz!

A nurse is providing information to a client immediately before their scheduled Romberg test. Which of the following statements should the nurse make?

"I will be checking you once with your eyes open and once with them closed." The nurse should inform the client that the Romberg test will be performed once with eyes open and once with eyes closed. A Romberg test is performed to assess balance and motor function. Incorrect The nurse should inform the client that, during the Romberg test, they will be standing with their feet together. The nurse should inform the client that, during the Romberg test, they will place their arms at their sides in a resting position. Standing across the room from the client during a Romberg test is a risk to the client's safety. Therefore, the nurse should stand close to the client to prevent the client from falling.

A nurse is teaching a client who has a new prescription for total parenteral nutrition through a central line. Which of the following information should the nurse include in the teaching? A. "I will change your IV tubing once every 48 hours." B. "Abdominal distention is an expected effect of this therapy." C. "I will need to check your gastric residual before administering feedings." D. "I will need to measure your weight daily."

"I will need to measure your weight daily." The nurse should instruct the client that daily weight measurement is a necessary part of administering nutrition through a central line to avoid fluid overload and monitor for adequate weight gain. Incorrect The nurse should change the client's IV tubing at least once every 24 hr to prevent bacterial growth in the tubing, which can lead to infection. Abdominal distention, along with nausea and vomiting, is a potential complication of overfeeding when the client is receiving feedings through a gastrointestinal tube. The nurse should check the gastric residual for a client who is receiving enteral nutrition to avoid overfeeding.

A charge nurse notices that one of the nurses on the shift frequently violates unit policies by taking an extended amount of time for break. Which of the following statements should the charge nurse make to address this conflict? A. "I would like to talk to you about the unit policies regarding break time." B. "If you continue to take a long lunch break, I will have to report this to the nurse manager." C. "Have you thought about how your extended lunch breaks affect the other members of our

"I would like to talk to you about the unit policies regarding break time." The charge nurse is dealing with the conflict in a cooperative, positive manner by using this statement to open the conversation in a nonthreatening way. The focus is on the length of the break time and is not a personal affront.

A nurse is caring for an adolescent client who has a new diagnosis of terminal cancer. When discussing the client's prognosis with the parents, the nurse should recognize which of the following responses by the parents as an example of rationalization? A. "Our child wouldn't have this terminal diagnosis if the doctor had diagnosed the cancer sooner." B. "Let's go on that family vacation we've got planned. We will deal with this when we return." C. "Maybe this is better for our child because we

"Maybe this is better for our child because we don't want any suffering through chemotherapy treatments." By justifying the adolescent's prognosis by searching for a more personally acceptable explanation for the impending loss, the parent is using the defense mechanism of rationalization. Incorrect By attributing the cause of the adolescent's prognosis to the provider's failure to diagnose the illness sooner, the parent is using the defense mechanism of displacement. By exhibiting a conscious denial of the adolescent's prognosis until the family returns from vacation, the parent is using the defense mechanism of suppression. By focusing on disbelieving the news about the adolescent's prognosis, the parent is using the defense mechanism of denial.

A nurse is teaching a client who has opioid use disorder about methadone. Which of the following information should the nurse include in the teaching? A. "If you suspect you are pregnant, stop taking this medication." B. "You cannot become physically dependent on this medication." C. "Sedation is a common adverse effect of this medication." D. "If you forget a dose, you can double your next dose."

"Sedation is a common adverse effect of this medication." Sedation and drowsiness are common adverse effects of methadone. Sedation most frequently occurs at the beginning of treatment or during dosage increases. Incorrect A client can take methadone to treat opioid withdrawal symptoms during pregnancy. A client can develop physical dependency with long-term use of methadone. Methadone can cause respiratory depression. The client should not take more than the prescribed dose at any time.

A nurse is caring for a client who is at 28 weeks of gestation. The client asks the nurse to explain what is causing the constipation. Which of the following responses should the nurse make? A. "Estrogen levels decrease during pregnancy, causing the stool to become hardened." B. "Decreased water absorption in the intestine during pregnancy causes constipation." C. "The intestine absorbs iron less efficiently during pregnancy, leading to constipation." D. "The enlarged uterus compresses the inte

"The enlarged uterus compresses the intestines and causes constipation." During the second and third trimesters, the size and weight of the growing uterus cause both displacement and compression of the intestines. These changes cause a decrease in motility, leading to constipation. Incorrect Estrogen and progesterone levels increase during pregnancy, leading to decreased peristalsis and relaxation of the smooth muscles of the intestine, which can result in constipation. The intestine absorbs more water from the stool during pregnancy, leading to constipation. The small intestine absorbs iron more readily during pregnancy due to increased maternal needs, leading to constipation.

A nurse is providing teaching to a client who is at 24 weeks of gestation and is scheduled for a 3-hr oral glucose tolerance test. Which of the following instructions should the nurse include in the teaching? A. "Limit your fat intake for 72 hours before the test." B. "You will need to fast the night before the test." C. "We will collect a urine sample the day after testing." D. "A blood sample will be collected every 15 minutes during the test."

"You will need to fast the night before the test." The nurse should instruct the client that they will need to fast the night before the test to prevent inaccurate test results. Incorrect The nurse should instruct the client to follow an unrestricted diet for 3 days prior to the test, which should include adequate carbohydrate intake of at least 150 g per day. Collecting a urine sample the day after the test will not provide information about the client's glucose tolerance. A urine sample can be collected at intervals during the test, but not after the test. The nurse should instruct the client that a blood sample will be collected every hour, rather than every 15 min, during the test.

A nurse is providing teaching to a parent of a child who has a permanent tracheostomy tube. Identify the sequence of steps the parent should follow to perform tracheostomy care.

- Remove the inner cancula - Remove the soiled dressing - Clean the stoma with 0.9% sodium chloride - Change the tracheostomy collar

A charge nurse is providing an educational session about infection control for a group of staff nurses. Which of the following statements by one of the staff nurses indicates an understanding of isolation precautions? A. "Droplet precautions should be initiated for a client who tests positive for measles." B. "A client who requires airborne precautions should be placed in a negative-pressure airflow room." C. "Airborne precautions should be initiated for a client who has Clostridium difficile

"A client who requires airborne precautions should be placed in a negative-pressure airflow room." Airborne precautions require a negative-pressure airflow room that has at least six to 12 air exchanges each hour using a HEPA filtration system. Incorrect The nurse should initiate airborne precautions, rather than droplet precautions, for a client who has measles. The nurse should initiate contact precautions, rather than airborne precautions, for a client who has C. difficile. The nurse should place a client who is immunocompromised in a protective environment, rather than a negative-pressure airflow room. A protective environment includes a private room that has positive airflow with at least 12 air exchanges each hour.

A nurse in a provider's office is assessing an adolescent who has been taking ibuprofen for 6 months to treat juvenile idiopathic arthritis. Which of the following questions should the nurse ask to assess for an adverse effect of this medication? A. "Have you experienced muscle stiffness?" B. "Have you had any stomach pain or bloody stools?" C. "Have you experienced a dry cough?" D. "Have you noticed an increase in urine output?"

"Have you had any stomach pain or bloody stools?" The nurse should ask the client about the presence of stomach pain or bloody stools, which is an indication of gastrointestinal bleeding, an adverse effect of ibuprofen. Incorrect Long-term ibuprofen use does not cause increased muscle stiffness. However, the client might have joint stiffness as a manifestation of juvenile idiopathic arthritis. Long-term ibuprofen use does not affect the respiratory system or cause a dry cough. The nurse should recognize that long-term ibuprofen use can damage the kidneys and assess the client for decreased urine output.

A nurse is admitting a client to the mental health unit after an attempted suicide. The client states, "My family does not care whether I live or die." Which of the following responses should the nurse make? A. "I'm sure your family does not want you to die." B. "Why would you believe such things?" C. "How does this make you feel?" D. "You should talk to your family about your feelings."

"How does this make you feel?" This response encourages the client to evaluate their feelings. Incorrect This response provides false reassurance and minimizes the client's feelings. This response criticizes the client and might result in the client feeling defensive. This response gives the client advice and inhibits problem-solving.

A nurse is providing discharge instructions to a client who has a new prescription for amitriptylline to treat depression. The nurse should identify that which of the following client statements indicates an understanding of the teaching? A. "I should avoid eating smoked meat, cheeses, and ripe avocados while taking this type of medication." B. "I should watch for common reactions like dry mouth and constipation." C. "I will be at increased risk for high blood pressure while taking this medic

"I should watch for common reactions like dry mouth and constipation." The nurse should reinforce that increasing dietary fiber, fluid intake, and chewing sugar-free gum can alleviate the anticholinergic effects of dry mouth and constipation. Incorrect A client who is taking an MAOI should avoid foods that contain tyramine. Clients who are taking amitriptyline should monitor for hypotension and change positions slowly. The client should take a daily dose of amitriptyline, a tricyclic antidepressant, at bedtime to promote sleep and minimize drowsiness during the day.

A nurse is conducting group therapy with clients who have breast cancer. The nurse should recognize which of the following statements by a client as an example of altruism? A. "I have experienced physical discomfort when intimate with my partner since my diagnosis." B. "I wish other people would stop socializing with my partner." C. "I told my doctor that I would like to start a support group for other people who are sick in my community." D. "I used to mistrust my doctor, but now I know the

"I told my doctor that I would like to start a support group for other people who are sick in my community." This statement indicates that the client is demonstrating altruism by reaching out and helping others.

A nurse is preparing to administer diazepam 0.3 mg/kg IV bolus to a toddler who weighs 10 kg (22 lb) and is experiencing a grand mal seizure. Available is diazepam solution for injection 5 mg/mL. How many mL should the nurse administer? (Round the answer to the nearest tenth.)

0.6 mL

A nurse is performing tracheostomy care for a client who is postoperative following a laryngectomy. Which of the following actions should the nurse take when suctioning the client's airway? A. Withdraw the catheter if the client begins coughing. B. Apply suction for 10 seconds. C. Advance the catheter 2 cm (0.8 in) after resistance is met. D. Use medical asepsis when performing the procedure.

Apply suction for 10 seconds. The nurse should apply suction for only 5 to 15 seconds to minimize oxygen loss. Incorrect Suctioning can initiate the cough reflex as it opens the airway further and allows for more effective removal of mucus. Once resistance is met, the nurse should withdraw the catheter 1 to 2 cm (0.4 in to 0.8 in) to prevent damaging bronchial tissues. The nurse should use surgical asepsis when suctioning a newly created tracheostomy to reduce the risk for infection.

An antepartum nurse is caring for four clients. For which of the following clients should the nurse initiate seizure precautions? A. A client who is at 33 weeks of gestation and has severe gestational hypertension B. A client who is at 16 weeks of gestation and has a hydatidiform mole C. A client who is at 28 weeks of gestation and is experiencing vaginal bleeding D. A client who is at 36 weeks of gestation and has a positive group B streptococcal culture

A client who is at 33 weeks of gestation and has severe gestational hypertension The nurse should initiate seizure precautions for a client who has severe gestational hypertension because an extremely elevated blood pressure in an antepartum client can trigger seizure activity. The nurse should provide the client with a quiet, darkened environment, place suction equipment and oxygen at the bedside, and position the call light within the client's reach. Incorrect A hydatidiform mole is a benign overgrowth of the chorionic villi that develops into cystic, grape-like clusters. Due to abnormal fertilization, there is no viable fetus in the uterus. Hemorrhage, rather than seizure activity, is the primary risk for a client who has a hydatidiform mole. Late pregnancy bleeding requires rapid assessment and treatment as it can indicate placenta previa or placental abruption. Hemorrhage, rather than seizure activity, is the primary risk for this client. A client who has group B streptococcus c

A nurse is providing teaching to the guardians of a newborn about measures to prevent sudden unexpected infant death (SUID). Which of the following guardian statements indicates an understanding of the teaching? A. "I will not allow anyone to smoke near my baby." B. "I will place bumper pads in my baby's crib." C. "My baby's head should be placed on a pillow for sleeping." D. "My baby should sleep in a side-lying position."

A. "I will not allow anyone to smoke near my baby." This statement by the guardian indicates an understanding of the nurse's instructions. Research indicates a strong correlation between exposure to cigarette smoke and the occurrence of SUID. Incorrect: The guardians should not place bumper pads in the infant's crib because they increase the risk for suffocation. Therefore, this is a risk factor for SUID. The guardians should not place the infant's head on a pillow for sleeping because it increases the risk for suffocation. Therefore, this is a risk factor for SUID. The guardians should place the child in a supine position for sleeping to prevent SUID.

The nurse is updating the plan of care for a client who is 48 hr postoperative following a laryngectomy and is unable to speak. Which of the following actions should the nurse plan to take first? A. Determine the client's reading skills. B. Instruct the client on esophageal speech technique. C. Provide the client with an alphabet board. D. Show the client how to use an artificial larynx.

A. Determine the client's reading skills. The first action the nurse should take when using the nursing process is to assess the client. By determining the client's level of reading skills and cognition, the nurse can best provide the client with a variety of customized techniques to practice and use after verbal skills are lost. Incorrect The nurse should instruct the client on the technique for esophageal speech and allow time for the client to practice. However, there is another action the nurse should take first. The nurse should provide the client with an alphabet board and demonstrate how to use it for communicating after verbal skills are lost. However, there is another action the nurse should take first. The nurse should show the client how to use an artificial larynx, called an electrolarynx, for communicating after verbal skills are lost following surgery. However, there is another action the nurse should take first.

A nurse is assessing an older adult client who has pneumonia. Which of the following findings should the nurse expect? A. Paradoxical chest movement B. Subcutaneous emphysema C. Acute confusion D. Distended neck veins

Acute confusion An older adult client who has pneumonia will also typically have acute confusion, fatigue, lethargy, and anorexia. Incorrect A client who has flail chest will have paradoxical chest movement. A client who has a pneumothorax will have subcutaneous emphysema. A client who has cor pulmonale or tension pneumothorax will have distended neck veins.

A nurse is providing teaching to a client who has a new diagnosis of type 1 diabetes mellitus. The nurse should instruct the client to monitor for which of the following findings as a manifestation of hypoglycemia? A. Irritability B. Increased urination C. Vomiting D. Facial warmth

Irritability The nurse should instruct the client to monitor for irritability, which can indicate decreased blood glucose levels. Incorrect Increased urination is an indication of hyperglycemia. Vomiting is an indication of hyperglycemia. Facial warmth is not a manifestation of hypoglycemia.

A nurse is planning morning care for a client who has heart disease and type 2 diabetes mellitus. Upon review of the client's medical record, which of the following actions should the nurse take? A. Give 4 oz of orange juice B. Administer daily medications C. Recheck the potassium level D. Contact the provider Medication Record Digoxin 0.25 mg PO dailyPotassium chloride 20 mEq/L PO dailyMetformin 500 mg PO dailyFurosemide 20 mg PO daily Vital Signs Blood pressure 116/62 mm HgRespiratory rate

Administer daily medications. The client's vital signs and laboratory data are within the expected reference range. Therefore, the nurse should administer the client's daily medications. Incorrect The client's blood glucose level is within the expected reference range. Therefore, it is not appropriate for the nurse to give the client 4 oz of orange juice. The client's potassium level is within the expected reference range. Therefore, it is not appropriate for the nurse to recheck this laboratory level. The client's vital signs and laboratory data are within the expected reference range. Therefore, it is not necessary for the nurse to contact the provider.

A nurse is caring for an older adult client who is experiencing chronic anorexia and is receiving enteral tube feedings. Which of the following laboratory values indicates the client needs additional nutrients added to the feeding? A. Creatinine 1.1 mg/dL (0.5 to 1.3 mg/dL) B. Albumin 2.8 g/dL (3.5 to 5 g/dL) C. Triglycerides 100 mg/dL (35 to 135 mg/dL) D. Alkaline phosphatase 118 units/L (30 to 120 units/L)

Albumin 2.8 g/dL (3.5 to 5 g/dL) The nurse should recognize that an albumin level of less than 3.5 g/dL indicates malnutrition and a need for additional nutritional supplementation. Incorrect A creatinine level of 1.1 mg/dL is within the expected reference range of 0.5 to 1.1 mg/dL for a female client, and 0.7 to 1.3 mg/dL for a male client. A triglyceride level of 100 mg/dL is within the expected reference range of 35 to 135 mg/dL for a female client, and 40 to 160 mg/dL for a male client. An alkaline phosphatase level of 118 units/L is within the expected reference range of 30 to 120 units/L. An elevated alkaline phosphatase level is an indication of liver or bone disorders, with a decreased level indicating malnutrition.

A nurse is caring for a client who has become aggressive and potentially violent. Which of the following actions should the nurse take? A. Allow the client time for reflection and decision making. B. Insist that the client behave appropriately. C. Use stern nonverbal communication. D. Remain within reach of the client.

Allow the client time for reflection and decision making. The nurse should allow the client silence and time to reflect on what is occurring as well as what decision they would like to make moving forward. Clients might feel more stressed if they feel they are being rushed, which can increase the chance of violent behavior. Incorrect The nurse should be cautious about insisting that the client do something. Instead, the nurse should allow the client to make decisions on activities that are negotiable. For example, the nurse should allow the client to choose a time to bathe instead of insisting that it be done at a certain time. The nurse should use nonverbal communication that is nonthreatening. The nurse's nonverbal communication should remain neutral. The nurse should stand at least 1 meter (3 feet) away from the client. This distance allows for personal safety of the nurse as well as respecting the client's feelings.

A nurse is caring for a client. Nurses' Notes Day 1, 1000: Client presents to the emergency department (ED) with right-sided hemiparesis, lethargy, and aphasia. The client's symptoms started 1 hr prior to arrival at the ED. Client received fibrinolytic therapy and was transferred to the ICU. Day 2, 0800: Client is awake and alert to person, place, and time. Client has weak right-side hand grasp. However, this is improved from admission. Client to be evaluated by speech therapy due to aphasia.

Anticipated: Administer oxygen therapy to keep oxygen saturation above 95%, Keep the lights in the client's room dim, Monitor blood glucose every 4 hr, Contraindicated: keep the client, supine, maintain the client's hips in flexion, cluster nursing care When generating solutions, the nurse should identify that oxygen therapy, monitoring blood glucose, and keeping lights in the client's room dim are anticipated prescriptions. The client is exhibiting manifestations of increased intracranial pressure (ICP). Therefore, the nurse should titrate oxygen therapy to maintain the oxygen saturation level above 95% and avoid hypoxia. The nurse should frequently monitor the client's vital signs and blood glucose to avoid secondary brain injury. The nurse should also dim the lights in the client's room, because many clients who have increased ICP experience photophobia.

A nurse is caring for a client following a laparoscopic cholecystectomy. Nurses' Notes 1030: 33-year-old client is 1 hr postoperative following a laparoscopic cholecystectomy. Alert and oriented to person, place, and time. Skin warm and dry. Lungs clear auscultated throughout all lung fields. Normal sinus rhythm. Client denies nausea and vomiting, bowel sounds hypoactive in all four quadrants. Peripheral pulses +2 bilaterally. Incision dressing clean and dry, incision intact upon inspection, no

Anticipated: change dressing when soiled, encourage deep breathing exercises every hour, apply heat for abdominal pas as needed Contraindicated: Ondansetron 4 mg PO for nausea When taking actions for a client who is postoperative following a laparoscopic cholecystectomy, the nurse should anticipate prescriptions for the client to apply heat for abdominal pain as needed, to encourage deep breathing, and to change the dressing when soiled. The client can use heat for abdominal pain related to carbon dioxide retention. During the procedure, carbon dioxide is inflated into the abdominal cavity for visualization for the provider. The client's dressing should be changed when soiled as needed. The dressing should be clean, dry, and intact to prevent infection. The nurse should identify that medication for nausea should be provided as needed and is contraindicated for scheduled administration.

A nurse is caring for a client who is taking valproic acid for seizure control. For which of the following adverse effects should the nurse monitor and report? A. Weight loss B. Jaundice C. Bradycardia D. Polyuria

Jaundice The nurse should monitor the client for jaundice and report any indication to the provider. Clients who take valproic acid are at risk for liver damage, which can lead to jaundice.

A nurse working on a medical-surgical unit receives a telephone call requesting the status of a client from an individual who identifies themselves as the client's guardian. Which of the following actions should the nurse take? A. Ask the caller for verification of their identity. B. Give the caller limited information about the client. C. Transfer the phone call to the client's room. D. Inform the caller that they should obtain permission from the client's provider.

Ask the caller for verification of their identity. According to HIPAA, if someone requests information about a client, it is the nurse's duty to protect that information. Therefore, the nurse should inform the caller that nurses cannot release any client information over the phone without the permission of the client. The nurse should ask for verification of the caller's identity to determine if they have been authorized by the client to receive information. Incorrect This action by the nurse violates HIPAA because the client's personal health information is protected legally by the facility and the nurse should not disclose information, even in a limited form. According to HIPAA, if someone requests information about a client, it is the nurse's duty to protect that information. Therefore, the nurse should inform the caller that they cannot transfer the call or release any client information over the phone without the permission of the client. The client's provider must receive permi

A nurse is caring for a client who has sensorineural hearing loss and is helping them choose items for their meal tray. Which of the following techniques should the nurse use to help the client communicate their choices? A. State the options loudly in a high-pitched voice. B. Expect extended time for verbal responses. C. Ask the client to point to items on a picture menu. D. Ask the client's partner to choose their meal.

Ask the client to point to items on a picture menu The nurse should recognize that using visual aids can help the client communicate their meal choices. The use of a visual aid, like a picture menu, can ensure the client understands the meal choices. Incorrect The nurse should recognize that clients who have sensorineural hearing loss have difficulty hearing high-frequency sounds. The nurse should speak in a lower tone of voice. The nurse should avoid speaking in a loud voice because it can make understanding what is being said more difficult. The nurse should recognize that clients who have a cognitive impairment may need additional time to comprehend and respond to meal choices. It is not an expected finding for clients who have sensorineural hearing loss to have difficulty with comprehension. The nurse should recognize that maintaining client autonomy is important. It is important for the nurse to assess the client's preferences for their preferred method of communication to allow

A nurse is caring for a client who had abdominal surgery 24 hr ago. Which of the following actions is the nurse's priority? A. Assess fluid intake every 24 hr. B. Ambulate three times a day. C. Assist with deep breathing and coughing. D. Monitor the incision site for findings of infection.

Assist with deep breathing and coughing. The priority action the nurse should take when using the airway, breathing, circulation approach to client care is to assist the client with deep breathing and coughing, which reduces the risk for postoperative pneumonia. Incorrect The nurse should assess the client's fluid intake every 24 hr to evaluate hydration status. However, another action is the nurse's priority. The nurse should ambulate the client three times a day to promote circulation. However, another action is the nurse's priority. The nurse should monitor the client's incision site for findings of infection. However, another action is the nurse's priority.

A nurse is caring for an older adult client in the PACU following general anesthesia. Which of the following findings should the nurse report to the provider? A. Urine output 120 mL in 4 hr B. Systolic blood pressure 12 mm Hg lower than the preoperative level C. Audible stridor D. Normal sinus rhythm with an occasional premature ventricular contraction

Audible stridor Audible stridor, or a high-pitched sound heard in the client's airway, indicates edema, laryngeal spasm, secretions, or some type of airway obstruction that could become life-threatening. The nurse should report this finding to the provider. Incorrect The nurse should monitor urinary output and report any amount less than 30 mL/hr. The nurse should report blood pressure changes that are greater than a 15 to 20 mm Hg difference from the client's baseline blood pressure. Anesthesia medications and surgery, especially in older adult clients, are common causes of premature ventricular contractions. The nurse should monitor the frequency of the premature ventricular contractions but does not need to report this finding to the provider.

A nurse manager is reviewing clients' rights with the nurses on the unit. The nurse manager should tell the nurses that informed consent promotes which of the following ethical principles? A. Autonomy B. Nonmaleficience C. Justice D. Fidelity

Autonomy Autonomy refers to a client's ability to make their own decisions about treatment. Informed consent promotes autonomy by providing clients with complete information about treatment. Incorrect Nonmaleficence is the duty to do no harm. Therefore, informed consent is not directly based on this ethical principle. Justice is the duty to treat everyone fairly. Therefore, informed consent is not directly based on this ethical principle. Fidelity is the duty to fulfill commitments. Therefore, informed consent is not directly based on this ethical principle.

A nurse is planning care for a client who is receiving chemotherapy and has neutropenia. Which of the following interventions should the nurse include in the plan? A. Avoid including raw fruits in the client's diet. B. Restrict visits from young children to 2 hr per day. C. Measure the client's temperature every 8 hr. D. Use disposable gloves from a box outside the client's room.

Avoid including raw fruits in the client's diet. The nurse should exclude raw fruits and vegetables from the client's diet to reduce the risk of bacterial infections. Incorrect The nurse should restrict the client's visitors to healthy adults to reduce the risk of infection. The nurse should measure the client's temperature every 4 hr to monitor for manifestations of an infection. The nurse should keep a box of disposable gloves in the client's room to reduce the risk of contamination.

A nurse is caring for an adolescent in the emergency department (ED) Laboratory Results Sodium 140 mEq/L (136 to 145 mEq/L) Potassium 3.6 mEq/L (3.5 to 5 mEq/L) Chloride 103 mEq/L (98 to 106 mEq/L) BUN 15 mg/dL (10 to 20 mg/dL) Magnesium 1.5 mEq/L (1.3 to 2.1 mEq/L) Total calcium 9.5 mg/dL (9 to 10.5 mg/dL) Phosphate 3.7 mg/dL (3 to 4.5 mg/dL) Glucose 80 mg/dL (74 to 106 mg/dL) Total protein 7 g/dL (6.4 to 8.3 g/dL) Albumin 4.5 g/dL (3.5 to 5 g/dL) WBC count 19,500/mm3 (5,000 to 10,000/mm3) Asp

Bacterial Meningitis: Temperature, photophobia, rash, mental status, and pain Encephalitis: Temperature, pain, and mental status Reye Syndrome: mental status and hepatic function When recognizing cues, the nurse should recognize that manifestations of bacterial meningitis can include fever, photophobia, nuchal rigidity, petechial rash, and impaired consciousness. The adolescent is experiencing these symptoms. Encephalitis is characterized by fever, nuchal rigidity, and altered mental status. Reye syndrome is characterized primarily by altered mental status and impaired hepatic function.

A nurse is assessing a client who has major depressive disorder and is taking amitriptylline. Which of the following findings should the nurse identify as an adverse effect of the medication? A. Diarrhea B. Frequent urination C. Excessive salivation D. Blurred vision

Blurred vision The nurse should identify blurred vision as an adverse effect of amitriptyline and notify the provider. Incorrect Constipation is an adverse effect of amitriptyline. Urinary retention is an adverse effect of amitriptyline. Dry mouth is an adverse effect of amitriptyline.

A nurse in a mental health clinic is assessing a client who has a history of seeking counseling for relationship problems. The client shows the nurse multiple superficial self-inflicted lacerations on their forearms. The nurse should identify these behaviors as characteristics of which of the following personality disorders? A. Borderline B. Antisocial C. Histrionic D. Paranoid

Borderline The nurse should identify that clients who have borderline personality disorder tend to be emotionally unstable, have troubled interpersonal relationships, and often engage in harmful behaviors such as cutting, substance use, and suicidal ideation. Incorrect Clients who have antisocial personality disorder tend to be manipulative, deceitful, and antagonistic. Criminal behavior and substance use are associated with this disorder. Clients who have histrionic personality disorder tend to be self-focused, melodramatic, attention-seeking, and impulsive. Typically, they do not believe they need mental health counseling. Clients who have paranoid personality disorder tend to be suspicious and distrustful of others. They display jealousy and expect hostility from others.

A nurse on a medical-surgical unit is caring for a client who has a new diagnosis of terminal cancer. The client tells the nurse that they would like to go home to be with family and loved ones. Which of the following actions should the nurse take? A. Contact the facility chaplain to visit with the client. B. Explain the process of leaving the facility against medical advice. C. Make a referral for social services. D. Encourage the client to continue with inpatient care.

C. Make a referral for social services. As a client advocate, the nurse should support the client's decisions and obtain a referral for social services to ensure that the client's needs at home are met. Social services can set up home care or hospice care services for the client if needed. Incorrect: The nurse should ask the client's permission before contacting the facility chaplain to visit. The nurse should identify that the client is not leaving the facility against medical advice. Therefore, the nurse should notify the provider of the client's wishes. The nurse should recognize the client's autonomy and support the client's wishes to go home.

The client returns to the provider's office 3 days later. Vital Signs​ Day 1, 0900: Temperature 37.2° C (99° F) Heart rate 88/min Blood pressure 142/88 mm Hg Respiratory rate 20/min Oxygen saturation 92% on room air 3 days later, 0900: Temperature 37.7° C (100.9° F) Heart rate 87/min Blood pressure 144/90 mm Hg Respiratory rate 22/min Oxygen saturation 93% on room air Nurses' Notes Day 1, 0900: Client is a 65-year-old who reports pain and burning on urination. Client states, "I am having

Click to highlight the findings that indicate the client's urinary tract infection is improving. To deselect a finding, click on the finding again. - Urine specific gravity - pH - WBC When evaluating outcomes, the nurse should identify that the client's urinary tract infection (UTI) is improving as evidenced by the client's urine specific gravity, pH, and WBC results. These findings are within the expected reference ranges and indicate that the medication has been effective in treating the UTI.

A nurse is assessing a newborn who is 3 days old History and Physical Newborn was delivered at 37 weeks of gestation via cesarean section for fetal distress. Apgar scores: 8 at 1 min and 9 at 5 min. Birth weight: 2.9 kg (6 lb 6 oz) The client who gave birth plans to breastfeed. Flow Sheet Day 2 of Life, 0900: Temperature 36.7° C (98.1° F) Heart rate 140/min Respiratory rate 48/min Weight 2.7 kg (6 lb); 6% weight loss Day 3 of Life, 0800: Temperature 36.4° C (97.5° F) Heart rate 140/min Res

Click to highlight the findings that require follow up. To deselect a finding, click on the finding again. - Temperature 36.4 C (97.5 F) - Weight 2.5 kg (5 lb 9 oz), 12% weight loss - Mild tremors noted when awake - Breastfeeding every 3 to 5 hr for 5 to 10 min. - Client reports nipple discomfort throughout the feeding When recognizing cues, the nurse should identify that a temperature of 36.4° C (97.5° F) is below the expected reference range. Hypothermia can lead to the occurrence of hypoglycemia and respiratory distress. The newborn breastfeeding for short intervals, nipple discomfort, and a weight loss of greater than 10% of birth weight can indicate inadequate transfer of breastmilk, which can result in hypoglycemia. The presence of mild tremors can be a manifestation of hypoglycemia.

A nurse is performing an admission assessment on a client who had a recent positive pregnancy test. The first day of their last menstrual period (LMP) was May 8. According to Naegele's rule, which of the following dates should the nurse document as the client's estimated date of birth (EDB)? February 1 February 8 February 15 February 22

February 15 Using Naegele's rule, the nurse should add 7 days to the first day of the client's LMP (8 + 7 = 15) and then subtract 3 months. Therefore, the nurse should document the client's EDB as February 15th.

A nurse is caring for a client on a medical-surgical unit Vital Signs 0700 Temperature 37.6 C (99.7 F) Heart rate 100/min Respiratory rate 22/min Blood pressure 115/70 mmHg Oxygen saturation 98% on room air Nurses' Notes 1100 Client alert and oriented to person, place, and time. Client had episode of diarrhea, provided perineal care. Noted 2 cm x 2 cm (0.8 in x 0.8 in) painful edematous area on sacrum. Client repositioned every 4 hr.

Click to highlight the findings that require follow up. To deselect a finding, click on the finding again. - Noted 2 cm x 2 cm (0.8 in x 0.8 in) painful edematous area on sacrum - Client repositioned every 4 hr When recognizing cues, the nurse should determine that the client's painful edematous area on their sacrum and that the client has only been repositioned every 4 hr requires follow up. The client has manifestations of a pressure injury that need to be addressed. The client should be repositioned at least every 2 hr to prevent worsening of the pressure injury and to relieve pressure from the sacral area.

A nurse on an antepartum unit is caring for a client who is at 33 weeks of gestation. Diagnostic Results WBC count 9,800/mm3 (5,000 to 10,000/mm3) Hgb 13 g/dL (greater than 11 g/dL) Hct 41% (greater than 33%) Platelet count 170,000/mm3 (150,000 to 400,000/mm3) BUN 20 mg/dL (10 to 20 mg/dL) Lactate dehydrogenase (LDH) 80 units/L (100 to 190 units/L) Aspartate aminotransferase (AST) 18 units/L (0 to 35 units/L) Alanine aminotransferase (ALT) 19 units/L (4 to 36 units/L) Uric acid (serum) 5.4 mg/d

Condition: Abruptio Placentae Actions: avoid cervical examination and insert a large-bore IV catheter Parameters to monitors: blood pressure and platelet count The nurse should avoid cervical examination and insert a large-bore IV catheter because the client is most likely experiencing abruptio placentae indicated by the sudden onset of abdominal pain, contractions, and dark red vaginal bleeding. Cervical examination can cause further damage to the placenta and increase bleeding. The nurse should immediately establish IV access with a large-bore catheter to administer IV fluids and blood products if bleeding increases or if manifestations of fetal distress occur. The nurse should monitor the client's blood pressure and platelet count because of the risk of significant blood loss due to the abruption. Hemorrhage might not be visible as vaginal bleeding if it is concealed between the placenta and uterine wall. Therefore, manifestations of hypovolemic shock (decreasing blood pressure, in

A nurse is caring for a 5-year-old child Physical Examination: 1510: Upon visual inspection, throat is inflamed, tonsils appear pink, reddened and epiglottis is edematous and cherry red in appearance. Skin appears pale. Stridor noted upon inspiration with diminished bilateral lung sounds. Nurse's Notes: 1500 Child accompanied to emergency department by caregiver. Caregiver states child has a sore throat and reports the child has "pain on swallowing" and denies cough. Child is agitated and lean

Condition: Epiglottis Actions: Initiate droplet precautions and request a prescription for IV antibiotics Monitors: Breath sounds and temperature The nurse should anticipate initiating droplet precautions and requesting a prescription for IV antibiotics. The child is most likely experiencing epiglottis because of the clinical manifestations of a high fever, inflammation and redness of the throat, pale skin, stridor with inspiration, painful swallowing, no cough, is sitting in tripod position, and drooling. The nurse should monitor the child's temperature and breath sounds.

A nurse is caring for a client who is postoperative following administration of general anesthesia. Vital Signs 0830: Temperature 36.9° C (98.5° F) Heart rate 134/min Respiratory rate 28/min Blood pressure 92/52 mm Hg Oxygen saturation 89% on room air Nurses' Notes 0830: Client is postoperative following an inguinal hernia repair.Apical pulse 134/min and irregular Client reports dyspnea. Diagnostic Results 0835: Arterial blood gases (ABGs) pH 7.30 (7.35 to 7.45) PCO2 64 mm Hg (35 to 45 mm

Condition: Malignant hyperthermia Actions: administer dantrolene and administer oxygen Parameters to monitor: Hypercapnia and muscle rigidity Upon recognizing and analyzing the client cues of tachycardia, tachypnea, hypotension, and irregular heart rhythm, the nurse's priority hypothesis should be that this client is most likely experiencing malignant hyperthermia and that it is important to generate solutions and take actions that will correct dysrhythmias, provide oxygen to tissues, correct electrolyte imbalances, and reverse metabolic and respiratory acidosis. Therefore, the nurse should prepare to administer dantrolene and administer oxygen. The nurse should monitor the PCO2 level on the client's ABGs for hypercapnia and observe the client for muscle rigidity of the jaw and chest muscles.

A nurse in an emergency department (ED) is assessing a client. Medical History 1030: Diagnosed with schizophrenia 2 years ago Migraine headaches Unresponsive to second-generation medications (clozapine and risperidone), changed to first-generation medication 6 months ago Current medications: Haloperidol 5 mg PO TIDSumatriptan 50 mg PO every 2 hr PRN headache Vital Signs 1030: Heart rate 122/minRespiratory rate 28/minBlood pressure 182/85 mm HgTemperature 39.7° C (103.5° F)Oxygen saturati

Condition: Neuroleptic malignant syndrome Action: hold the client's antipsychotic medications and apply a cooling blanket Monitor: temperature and hydration status Upon recognizing and analyzing the client cues of decreased responsiveness, muscle rigidity, posturing, diaphoresis, and vital signs that are outside the expected reference ranges, the nurse's priority hypotheses should be that this client is most likely experiencing neuroleptic malignant syndrome, which is related to the client's haloperidol therapy. It is important to generate solutions and take actions that will decrease the client's temperature, blood pressure, heart rate, and respiratory status, which will improve the client's neurological status. The nurse should hold the client's antipsychotic medications and apply a cooling blanket to reduce the client's temperature. Neuroleptic malignant syndrome is a life-threatening condition. Therefore, the nurse should monitor the client's laboratory and arterial blood gas valu

A nurse is caring for a client who states, "My boss accused me of stealing yesterday. I was so angry I went to the gym and worked out." The nurse should recognize the client is demonstrating which of the following defense mechanism? A. Displacement B. Regression C. Suppression D. Sublimation

Sublimation The client is exhibiting behaviors consistent with sublimation, which is displayed when a client substitutes socially unacceptable behavior for acceptable behavior. Incorrect Suppression is the denial of a disturbing feeling or situation. Regression occurs when a client reverts to a childlike pattern of behavior that might have been exhibited previously. Displacement occurs when a client transfers emotions of a particular situation to another nonthreatening situation.

A nurse manager is preparing an educational session for nursing staff about how to provide cost-effective care. Which of the following methods should the nurse include in the teaching? A. Delegate non-nursing tasks to ancillary staff. B. Stock client rooms with extra supplies. C. Assign dedicated equipment to each client's room. D. Change continuous IV infusion tubing every 24 hr.

Delegate non-nursing tasks to ancillary staff. Delegating non-nursing tasks to ancillary staff is an effective method of providing high-quality, cost-effective care because this will allow additional time for nurses to focus on skilled tasks. Incorrect Stocking client rooms with extra supplies is an ineffective method of providing cost-effective care. Client care supplies are direct costs that fluctuate with the volume of service. Using this method, it is difficult to keep up with inventory and the facility might charge clients for supplies they do not use. Assigning dedicated equipment to each client's room is an ineffective method of providing cost-effective care. Clients who have infectious diseases, such as methicillin-resistant Staphylococcus aureus, should have dedicated equipment. Providing dedicated equipment for all clients would unnecessarily increase the direct costs for each client on the unit. Changing continuous IV infusion tubing no more than every 96 hr is an effectiv

A nurse is interviewing a client who is now without a home due to a natural disaster. After ensuring the client's safety, which of the following actions should the nurse take first? A. Assist the client with contacting individuals from the client's support system. B. Give the client information about available community resources for shelter. C. Suggest the client obtain mental health counseling. D. Determine the client's perception of the personal impact of the crisis.

Determine the client's perception of the personal impact of the crisis. The first action the nurse should take using the nursing process is to assess the client. Therefore, the nurse should first determine the client's feelings and understanding of the natural disaster and its personal impact. Incorrect The nurse should assist the client with contacting people for support and additional help. However, there is another action that the nurse should take first. The nurse should assist the client with locating community resources to secure shelter. However, there is another action that the nurse should take first. The nurse should assist a client who recently experienced a natural disaster to obtain mental health counseling for further support. However, there is another action that the nurse should take first.

A nurse in an emergency department is preparing to discharge a client who has experienced intimate partner violence. Which of the following actions should the nurse take first? A. Offer a referral to the client for social services. B. Develop a safety plan with the client. C. Encourage the client to reach out to family and friends. D. Provide the client with a list of support groups.

Develop a safety plan with the client. The greatest risk to this client is injury from violence. Therefore, the first action the nurse should take is to develop a safety plan with the client. Incorrect A client who has experienced intimate partner violence can benefit from a referral to social services, but offering a referral is not the first action the nurse should take. The client can benefit from the support of family and friends when the client is ready, but encouraging this behavior is not the first action the nurse should take. The client can benefit from attending a support group, but providing this information is not the first action the nurse should take.

A nurse on a medical-surgical unit is assessing a client who has had a stroke. For which of the following findings should the nurse initiate a referral for occupational therapy? A. Difficulty performing ADLs B. Inability to swallow clear liquids C. Elevated blood glucose levels D. Unsteady gait when ambulating

Difficulty performing ADLs The nurse should initiate a referral for occupational therapy to teach the client the skills necessary to become independent in performing ADLs such as bathing, dressing, or eating. Incorrect The nurse should initiate a referral to a speech-language pathologist for a client who has any swallowing difficulties. The nurse should initiate a referral to a dietitian for a client who has elevated blood glucose levels. A dietitian provides support and teaching to the client to ensure nutritional needs are met. The nurse should initiate a referral for physical therapy for a client who has an unsteady gait when ambulating. A physical therapist develops a plan to improve a client's strength and mobility and can identify assistive devices that might be necessary.

A mental health nurse is conducting the first of several meetings with a client whose partner recently died. The nurse should perform which of the following actions to establish trust during the orientation phase of the nurse-client relationship? A. Encourage the client's problem-solving abilities. B. Discuss the client's previous experience with loss. C. Promote the client's self-esteem. D. Establish the termination date of therapy.

Establish the termination date of therapy. This task occurs in the orientation phase of a therapeutic relationship. Incorrect This task occurs in the working phase of a therapeutic relationship.

A nurse is preparing to transfer a client who has had a stroke to a rehabilitation facility. The client's family tells the nurse they are concerned about the level of care the client will receive. Which of the following actions should the nurse take? A. Facilitate an interdisciplinary conference at the new facility for the family. B. Refer the client and family to a social worker for assistance and a follow-up meeting. C. Reassure the client's family that the same provider will provide care a

Facilitate an interdisciplinary conference at the new facility for the family. Initiating an interdisciplinary conference will address the family's concerns about providing optimal care for the client. Incorrect There is no indication that the family needs a referral to a social worker, who assists families with financial issues and psychosocial needs. This action does not address the concerns of the client's family. This information provides false reassurance to the family. There is no guarantee that the same provider will be able to care for the client at the new facility. This information provides false reassurance to the family.

A nurse is caring for a client who has active tuberculosis (TB). Which of the following actions should the nurse plan to take to prevent the transmission of the disease? A. Initiate contact precautions for the client upon admission. B. Restrict visitors from entering the client's room during hospitalization. C. Wear a surgical mask while providing care for the client. D. Have the client wear a surgical mask while being transported outside the room.

Have the client wear a surgical mask while being transported outside the room. A client who has active TB should wear a surgical mask while being transported to prevent transmission of the disease. Incorrect The nurse should initiate airborne precautions, rather than contact precautions, for a client who has active TB. Mycobacterium tuberculosis is transmitted by air in nuclei smaller than 5 microns. Restricting visitors from entering the client's room is unnecessary if proper transmission precautions are taken. The nurse should wear an N95 or HEPA respirator when caring for a client who has active TB.

A nurse is caring for a client who has abdominal pain Nurses' Notes 0900 Client reports loss of appetite, weight loss, and fatigue for 1 week. Reports abdominal pain, 6 on a scale from 0 to 10, for 2 days. Client is a perioperative nurse, returned 1 week ago from a 2-week mission trip to an underdeveloped country 1200 Results of antibody studies obtained. Provider prescription for antiviral medication pending. Physical Examination 0930 Lung sounds clear bilaterally. Skin warm to touch and jau

Hepatitis A: Client's risk from fecal-oral transmission, laboratory results, and physical examination findings Hepatitis B: Antiviral treatment, laboratory results, client's risk from bloodborne transmission, physical examination findings Hepatitis C: Antiviral treatment, laboratory results, client's risk from bloodborne transmission, and physical examination findings When analyzing cues, the nurse should recognize that manifestations of hepatitis A, hepatitis B, and hepatitis C include jaundice, yellow sclerae, right upper quandrant pain upon palpation, dark yellow urine, and elevated AST and ALT levels. When analyzing cues, the nurse should also recognize the client's risk for contracting hepatitis A through the fecal-oral route during recent travel to an underdeveloped country and the client's occupational risk as a perioperative nurse for contracting hepatitis B and hepatitis C through bloodborne transmission. The nurse should recognize that the current standard of practice for

A nurse is caring for a client who requires physical therapy following discharge. Which of the following actions should the nurse take? A. Initiate the referral at the time of discharge. B. Have the client contact a physical therapist when they are ready to begin therapy. C. Verify that insurance will pay for outpatient physical therapy. D. Involve the client in selection of a physical therapy provider.

Involve the client in selection of a physical therapy provider. The nurse should involve the client in the referral process, including selection of the physical therapist and the location. Incorrect The nurse should initiate the referral as soon as possible after identifying the need. Waiting until the time of discharge can delay the client's recovery. Instructing the client to contact a physical therapist when they feel ready can significantly delay recovery. The nurse should initiate the referral as soon as possible after receiving a prescription from the provider. The nurse should notify the case manager or social worker of the prescription for physical therapy. They will search for providers that are willing to take the client's insurance and report to the nurse which facilities the client can consider. The client can then choose from that selection of providers.

A charge nurse is preparing to administer 0900 medications and is told by the pharmacy staff that the medications are not available. Medication availability has been an ongoing problem, and the charge nurse has previously discussed this issue with the pharmacy staff. Which of the following actions should the charge nurse take first? A. Document the actual time of medication administration. B. Notify the risk manager. C. Complete an incident report. D. Inform the nurse manager of the issue.

Inform the nurse manager of the issue. The greatest risk to clients is injury from not receiving medications on time and developing a medical complication. Therefore, the priority intervention the charge nurse should take is to follow the chain of command and contact the nurse manager. Incorrect The charge nurse should ensure nurses document the actual time of medication administration to maintain each client's medical administration record. However, there is another action that the charge nurse should take first. The charge nurse should notify the risk manager, who will develop an interprofessional team to improve the medication delivery process by gathering information about the situation. However, there is another action that the charge nurse should take first. The charge nurse should complete an incident report of the occurrence to make all facts about the incident available to the health care facility. However, there is another action that the charge nurse should take first.

A nurse is caring for a client who has generalized anxiety disorder and is to begin taking alprazolam. Which of the following actions should the nurse take? A. Check the client's temperature every 2 hr. B. Initiate fall precautions for the client. C. Monitor the client's urine for discoloration. D. Limit the client's fluid intake to 1 L per day.

Initiate fall precautions for the client. The nurse should initiate fall precautions for a client who has a new prescription for alprazolam because common adverse effects associated with this medication are orthostatic hypotension, dizziness, confusion, and lethargy. Incorrect Alprazolam does not affect temperature regulation. Therefore, monitoring the client's temperature as often as every 2 hr is not necessary. Urine discoloration is not an adverse effect of alprazolam. Therefore, monitoring the client's urine is not necessary. The nurse should encourage the client to increase fluid intake while taking alprazolam because an adverse effect of this medication is constipation.

A nurse is providing discharge instructions to a client following a total hip arthroplasty. Which of the following instructions should the nurse include? A. Minimize the use of a walker B. Maintain the hip at an angle greater than 90°. C. Install a raised toilet seat at home. D. Place a pillow under the knees when lying down.

Install a raised toilet seat at home. The client should use a raised toilet seat at home to minimize hip flexion and prevent hip dislocation. Incorrect The client should use a walker to minimize the risk of falls or injury. The client should maintain the hip at an angle less than 90° when sitting to minimize hip flexion and prevent hip dislocation. The client should not have a pillow under the knees when lying down because it can impede circulation and result in flexion contractures.

A nurse is caring for a client who has a deep vein thrombosis. Which of the following actions should the nurse take? A. Teach the client to massage the affected extremity. B. Instruct the client to elevate the affected extremity when sitting. C. Assess pulses proximal to the affected area. D. Apply a cold compress to the affected extremity.

Instruct the client to elevate the affected extremity when sitting. The nurse should instruct the client to elevate the affected extremity when in the bed or chair. Incorrect Massaging the affected extremity increases the risk of dislodging the thrombus and causing an embolus that can travel to the lungs. The nurse should assess pulses distal to the affected area. The nurse should apply warm, moist compresses to the affected extremity.

A nurse is preparing a client for a parecentesis. Which of the following actions should the nurse take? A. Instruct the client to void. B. Position the client on their left side. C. Insert an IV catheter. D. Prepare the client for moderate (conscious) sedation.

Instruct the client to void. The nurse should instruct the client to void prior to the procedure because an empty bladder decreases the risk of a bladder puncture and minimizes the client's discomfort during the procedure. Incorrect The nurse should position the client upright or in high-Fowler's position. An IV catheter is not necessary for a paracentesis. The nurse should instruct the client that the provider will administer a local anesthetic before beginning the procedure. Moderate sedation is not required for a paracentesis.

A nurse is providing dietary teaching to the parents of a 6 month-old infant. Which of the following instructions should the nurse include? A. Provide the infant with 1 cup of cereal. B. Give the infant 240 mL (8 oz) of juice per day. C. Introduce new foods one at a time over 3 to 5 days. D. Give whole milk first, then small amounts of solid food.

Introduce new foods one at a time over 3 to 5 days. The parents should introduce new foods one at a time over 3 to 5 days to identify potential food allergies. Incorrect Infants' portion sizes in general should be 1 Tbsp per year of age. For infants under 12 months of age, ½ to ¾ Tbsp is appropriate. The parents should offer the infant 100% fruit juice, not to exceed 120 to 180 mL (4 to 6 oz) per day, after 6 months of age. The parents should not offer the infant whole milk, because the majority of the infant's calories should come from human milk or commercial, iron-fortified formula.

A nurse manager is preparing a newly licensed nurse's performance appraisal. Which of the following methods should the nurse manager use to evaluate the nurse's time management skills? A. Compare the nurse's time management skills to the skills of coworkers. B. Review client satisfaction reports about the nurse's performance. C. Ask another staff nurse to evaluate the nurse's time management skills. D. Maintain regular notes about the nurse's time management skills.

Maintain regular notes about the nurse's time management skills. Maintaining notes over a period of time provides a comprehensive view of the nurse's abilities, so the manager can identify trends in the nurse's overall performance. Incorrect The nurse manager should compare the nurse's performance to standards from the nurse's job description to avoid potential bias. Client satisfaction reports concerning clients' facility stay are completed and evaluated to ensure that the client received quality care and reflect the performance of all members of the interdisciplinary team. It is not the responsibility of other staff nurses to provide the nurse manager with evaluative data about a coworker's time management skills. The nurse manager should gather data from other individuals who supervise the nurse, such as a charge nurse or unit supervisor, and have the nurse complete a self-evaluation.

A nurse is assessing a school-age child who has bacterial meningitis. Which of the following findings should the nurse expect? A. Nuchal rigidity B. Weight gain C. Tinnitus D. Positive Trendelenburg sign

Nuchal rigidity Nuchal rigidity is a manifestation of bacterial meningitis. Incorrect Weight loss is a manifestation of bacterial meningitis. Tinnitus is not a manifestation of bacterial meningitis. A positive Trendelenburg sign is a manifestation of developmental dysplasia of the hip and is not a manifestation of bacterial meningitis.

A nurse is caring for a client who has had nausea and vomiting for the past 2 days. The nurse should identify which of the following findings as an indication the client is experiencing fluid volume deficit? A. Shortness of breath B. Visual disturbances C. Decreased BUN levels D. Orthostatic hypotension

Orthostatic hypotension Clients who have a fluid volume deficit can experience orthostatic hypotension, which is a result of the body's inability to maintain adequate blood pressure following position changes. Incorrect The nurse should identify shortness of breath as an indication of fluid volume excess because extra fluid interferes with oxygen exchange at the alveolar level. The nurse should identify that visual disturbances, such as blurred vision, indicate fluid overload rather than fluid volume deficit. ncreased BUN levels should indicate to the nurse that the client has a fluid volume deficit.

A nurse is assessing a client following a vaginal delivery and notes heavy lochia and a boggy fundus. Which of the following medications should the nurse expect to administer? A. Nalbuphine B. Terbutaline C. Oxytocin D. Magnesium sulfate

Oxytocin The nurse should administer oxytocin, a hormone that stimulates uterine contractions, to decrease vaginal bleeding. Incorrect The nurse should administer nalbuphine, an opioid agonist-antagonist, for pain relief during labor. The nurse should administer terbutaline, a smooth muscle relaxant, to decrease contractions during preterm labor. The nurse should administer magnesium sulfate, a smooth muscle relaxant, to prevent seizures in a client who has preeclampsia.

A nurse is caring for a client who is at 37 weeks of gestation and is experiencing abruptio placentae. Which of the following findings should the nurse expect? A. Persistent uterine contractions B. Bright red vaginal bleeding C. Hyperactive deep-tendon reflexes D. Fundal height of 40 cm (15.7 in)

Persistent uterine contractions The nurse should expect a client who has abruptio placentae to experience persistent uterine contractions, board-like abdomen, and dark red vaginal bleeding. Incorrect The nurse should expect a client who has placenta previa to experience a relaxed uterus and bright red vaginal bleeding. With abruptio placentae, the nurse should expect to find dark red vaginal bleeding. The nurse should expect a client who has preeclampsia to have hyperactive deep-tendon reflexes. The nurse should expect a client who has placenta previa to have a fundal height that is greater than expected gestational age.

A nurse is assessing a client who has pulmonary edema. Which of the following findings should the nurse expect? A. Pink, frothy sputum B. Bradycardia C. Pale, dry skin D. Wheezing

Pink, frothy sputum A client who has manifestations of pulmonary edema can have pink, frothy sputum due to fluid leaking across the pulmonary capillaries and into the lung tissue. Incorrect A client who has manifestations of pulmonary edema will have tachycardia due to insufficient oxygen exchange and perfusion caused by fluid in the lung tissue. A client who has manifestations of pulmonary edema will have clammy, cyanotic skin due to insufficient oxygen exchange and perfusion caused by fluid in the lung tissue. A client who has manifestations of pulmonary edema can have crackles due to fluid in the lung tissue. Crackles can progress as the condition worsens.

A nurse is providing colostomy care for a client using a two-piece pouching system. Which of the following actions should the nurse take? A. Cleanse the skin at the stoma site with povidone-iodine for 15 seconds. B. Dampen the skin before applying the skin barrier and ostomy pouch. C. Place the skin barrier over the stoma and hold it for 30 seconds. D. Cut the skin barrier opening 0.6 cm (0.24 in) larger than the stoma.

Place the skin barrier over the stoma and hold it for 30 seconds. The nurse should activate the adhesive in the skin barrier by holding it in place over the stoma for 30 seconds. Incorrect The nurse should cleanse the skin at the stoma site using a washcloth and warm water to reduce the risk of skin irritation. The nurse should thoroughly dry the skin around the stoma using a patting motion before applying the skin barrier to ensure the pouch adheres to the client's skin. The nurse should cut the skin barrier opening no more than 0.3 cm (0.12 in) larger than the stoma to reduce the risk of skin irritation.

A nurse is caring for a client who is immediately postoperative following a subtotal thyroidectomy. Vital Signs 1100: Temperature 37.4° C (99.4° F) Heart rate 98/min Respiratory rate 18/min Blood pressure 128/68 mm Hg Oxygen saturation 97% on room air 1115: Temperature 37.8° C (100.1° F) Heart rate 110/min Respiratory rate 16/min Blood pressure 138/74 mm Hg Pulse oximetry 95% on room air 1130: Temperature 38.6° C (101.5° F) Heart rate 136/min Respiratory rate 16/min Blood pressure 154

Select the 4 client findings that lead the nurse to suspect that the client is experiencing thyroid storm. - Mental status - Heart rate - Temperature - Blood pressure When analyzing cues, the nurse should identify that thyroid storm can be caused by trauma to the thyroid gland, such as surgery, and excessive release of thyroid hormone greatly increases the metabolic rate. Fever greater than 38.5° C (101.3° F), heart rate greater than 130/min, systolic hypertension, and mental status changes, such as confusion, restlessness, and sleepiness, are characteristic of thyroid storm.

A nurse is caring for a client at a provider's office. History and Physical 2 months ago: Client presented to clinic for routine visit. Client reported feeling tired at times but getting through the workday and walking after work. Reported chronic nonproductive cough. Smokes 1.5 packs of cigarettes per day. Today, 1030: Client reports fatigue over the past several days, spending more time in bed. Reports chronic productive cough with blood-tinged sputum this morning. Smokes 1 pack of cigarett

Pneumonia: tobacco use, WBC level, Temperature, oxygen saturation, ABG results COPD: tobacco use and oxygen saturation Heart Failure: tobacco use, BNP level and oxygen saturation The nurse should analyze cues of pneumonia that include tobacco use, elevated WBC count, a productive cough with blood-tinged sputum, elevated temperature, a decreased oxygen saturation level, and an ABG level indicating respiratory acidosis. The nurse should also analyze cues of COPD that include tobacco use and a decreased oxygen saturation. The nurse should also analyze cues of heart failure that include tobacco use, BNP level, and a decreased oxygen saturation.

A nurse is administering 1 unit of packed RBCs to a client. The client becomes anxious and reports shortness of breath and urticaria 15 min after initiation of the transfusion. Which of the following actions should the nurse take? A. Prepare to administer epinephrine to the client. B. Decrease the rate of the client's transfusion. C. Obtain a culture of the client's blood. D. Anticipate administering diuretics to the client.

Prepare to administer epinephrine to the client. The nurse should recognize that the client is experiencing an anaphylactic reaction to the blood transfusion. Therefore, the nurse should prepare to administer epinephrine to the client to alleviate manifestations of anaphylaxis. Incorrect The nurse should stop the transfusion for a client who is experiencing an anaphylactic reaction to a blood transfusion. The nurse should obtain a culture of the client's blood for a client who has obtained a bacterial infection from contaminated blood, which can cause sepsis. This determines the bacteria that is to be treated with antibiotics. The nurse should anticipate administering diuretics to a client who is experiencing a circulatory overload reaction to a blood transfusion. This can assist with alleviating the additional fluid from the client's body.

A nurse is caring for a client who is pregnant. Nurses' Notes 1000: The client reports repeated episodes of vomiting and two episodes of diarrhea in past 24 hr. Client is at 18 weeks of gestation and reports a history of nausea and vomiting for the past 12 weeks. 1015: IV fluids initiated. Prochlorperazine administered via intermittent IV bolus. 1100: Client reports improvement in nausea. Ice chips provided. Client voided 50 mL of dark yellow urine. 1500: Client tolerating fluids well. Ate

Recommended: Alternate eating solid foods and liquids, eat every 2 to 3 hours, drink warm ginger ale when nauseated Contraindicated: Increase intake of high-fat foods When taking action and providing discharge teaching for a client who has hyperemesis gravidarum, the nurse should recommend the client should eat every 2 to 3 hr to avoid having an empty stomach, which can increase nausea. The client should separate liquids from solids every 2 to 3 hr to help minimize nausea. The client should eat foods high in protein that are low in fat. Warm ginger ale or ginger tea can also decrease nausea.

A nurse is assessing a client after administering epinephrine for an anaphylactic reaction. Which of the following findings should the nurse identify as an adverse effect of this medication? A. Hypotension B. Report of tinnitus C. Report of chest pain D. Ecchymosis

Report of chest pain The nurse should identify that a report of chest pain by the client can indicate an adverse effect of the medication. Epinephrine increases cardiac workload and oxygen demand, which can result in angina. Incorrect Hypertension is an adverse effect of epinephrine due to the vasoconstrictive actions of epinephrine. Tinnitus is not an adverse effect of epinephrine. Ecchymosis is not an adverse effect of epinephrine.

A nurse in an outpatient mental health clinic is caring for a client Vital Signs 3 months ago Blood pressure 116/68 mmHg Heart rate 82/min Respiratory rate 16/min Temperature 36.7 C (98.1 F) SaO2 97% on room air Today: Blood pressure 128/76 mmHg Heart rate 104/min Respiratory rate 22/min Temperature 37.4 (99.4 F) SaO2 97% on room air Nurses' Notes 3 months ago Client recently admitted with new diagnosis of schizophrenia. Received inpatient treatment for 10 days and was discharged 1 week ago.

Select the 3 findings that require immediate follow up: - Auditory hallucinations - Speech - Restlessness When recognizing cues, the nurse should identify that the findings of restlessness, auditory hallucinations, and pressured speech require immediate follow up. These findings are indications of psychosis. The nurse should notify the provider for additional evaluation and treatment.

A nurse in an acute mental health facility is planning care for a client who has anorexia nervosa. Which of the following interventions should the nurse include in the client's plan of care? A. Give the client a choice of foods and beverages. B. Supervise the client during and after eating. C. Encourage casual conversation about food during mealtimes. D. Provide opportunities for the client to choose their own mealtimes.

Supervise the client during and after eating. The nurse should monitor the client during and for 1 hr after meals to prevent the client from hiding food or purging. Incorrect The nurse should offer the client a structured meal plan to ensure appropriate caloric intake and adequate nutrition. The nurse should encourage conversation that does not focus on the theme of food during mealtimes. The nurse should emphasize eating as a social activity. The nurse should establish specific mealtimes as part of a structured meal plan.

A school nurse is notified of an emergency in which several children were injured following the collapse of playground equipment. Upon arrival at the playground, which of the following actions should the nurse take first? A. Instruct a staff member to maintain a log of emergency care provided. B. Apply cervical spine collars to children who have suspected neck trauma. C. Notify guardians of the emergency and injuries to their children. D. Survey the scene for potential hazards to staff and c

Survey the scene for potential hazards to staff and children. The first action the nurse should take when using the nursing process is to assess the situation. By surveying the scene, the nurse can identify potential hazards to staff and children. These findings allow the nurse and staff to enter the scene and safely provide care to injured children and help decrease the risk for further injury. Incorrect Instructing a staff member to maintain a log of emergency care provided ensures that documentation is available for interprofessional communication and keeps the nurse free to provide care to injured children. However, there is another action the nurse should take first. The nurse should apply cervical spine collars to children who have suspected neck trauma to immobilize the cervical spine and reduce the risk for further injury. However, there is another action the nurse should take first. The nurse should assist with notification of guardians following an emergency to ensure that

A nurse is providing teaching to a client who has a prescription for levothyroxine 25 mcg PO daily. Which of the following instructions should the nurse include in the teaching? A. Take the medication on an empty stomach 30 min before breakfast. B. Take the medication 2 hr apart from calcium supplements. C. Take a double dose of the mediation if the last dose was missed. D. Take this medication with a small sip of water.

Take the medication on an empty stomach 30 min before breakfast. The nurse should recognize that levothyroxine should be administered 30 to 60 min before breakfast to optimize absorption of the medication. Incorrect Levothyroxine should be taken 4 hr apart from calcium supplements to optimize absorption of the medication. If a dose is missed, the client should take it as soon as possible. The client should avoid double dosing the medication. Levothyroxine can cause choking and gagging. Therefore, it is recommended to drink a full glass of water when taking this medication.

A nurse is caring for a client who is postoperative following coronary artery bypass surgery (CABG) Laboratory Results 0630 Sodium 145 mEq/L (136 to 145 mEq/L) Potassium 3.2 mEq/L (3.5 to 5 mEq/L) Chloride 116 mEq/L (98 to 106 mEq/L) BUN 24 mg/dL (10 to 20 mg/dL) Magnesium 1.5 mEq/L (1.3 to 2.1 mEq/L) Total calcium 9 mg/dL (9 to 10.5 mg/dL) Phosphate 4.6 mg/dL (3 to 4.5 mg/dL) Glucose 95 mg/dL (74 to 106 mg/dL) WBC count 9,500/mm3 (5,000 to 10,000/mm3) I & O 0700 4 hr input 400 mL 4 hr output

The client is at greatest risk for developing dysrhythmias, as evidenced by electrolyte imbalance. The nurse should analyze cues to determine the client is at greatest risk for developing dysrhythmias related to hypokalemia, as evidenced by the laboratory report and the client's report of muscle cramping. Potassium and magnesium depletion are common manifestations in clients who are postoperative following CABG. Due to medication or hemodilation, it is important for the nurse to closely monitor electrolytes.

A nurse is caring for a client who is 1 day postoperative following a total thyroidectomy Laboratory Results 0700: Sodium 143 mEq/L (136 to 145 mEq/L) Potassium 3.5 mEq/L (3.5 to 5 mEq/L) Chloride 104 mEq/L (98 to 106 mEq/L ) BUN 15 mg/dl (10 to 20 mg/dl) Magnesium 1.5 mEq/L (1.3 to 2.1 mEq/L ) Total calcium 8 mg/dL (9 to 10.5 mg/dL) Phosphate 4.6 mg/dL (3 to 4.5 mg/dL) Glucose 95 mg/dL (74 to 106 mg/dL) WBC 9,500/mm3 (5,000 to 10,000/mm3) Nurses' Notes 0700: Client alert and oriented to perso

The client is at highest risk for developing hypocalcemia, as evidenced by the report of numbness around lips. The nurse should recognize cues and determine that the client is at highest risk for developing hypocalcemia as evidenced by the client's report of muscle spasms, numbness around lips, and decreased calcium level. Hypocalcemia is more likely to occur in clients who have experienced a thyroidectomy, due to accidental damage to the parathyroid. Numbness around the lips is a clinical manifestation specific to hypocalcemia. Hypocalcemia presents as muscle spasms and can lead to cardiac dysrhythmias. Hypocalcemia is the highest priority, as it requires immediate treatment with calcium gluconate to avoid dysrhythmias and other complications.

A nurse in a provider's office is caring for a client. Provider Prescriptions Day 1, 0930: Collect urine specimen for urinalysis and urine culture and sensitivity. Trimethoprim/sulfamethoxazole 160/800 mg PO twice daily for 10 daysPhenazopyridine 200 mg PO every 6 hr for 2 days Laboratory Results Day 1, 1100: UrinalysisColor: Amber (Amber yellow)Appearance: Cloudy (Clear) Specific gravity: 1.04 (1.005 to 1.03)pH: 9 (4.6 to 8)Glucose: None (None)Ketones: None (None)Bilirubin: None (None)Blood:

The client is at highest risk for developing pyelonephritis, as evidenced by the client's urinalysis results The nurse should determine that the priority hypothesis is the client is at the highest risk for developing pyelonephritis as evidenced by the client's urinalysis results. The urinalysis indicates dark cloudy urine, increased specific gravity, increased pH, increased red and white blood cells, positive nitrites, positive leukocytes, and trace amounts of blood, which indicate a urinary tract infection (UTI). If left untreated, a UTI can lead to pyelonephritis.

A nurse on the medical-surgical unit is caring for a client who was admitted from the emergency department (ED) Vital Signs 1400 Temperature 38 C (100.4 F) Heart rate 110/min Respiratory rate 24/min Blood pressure 96/58 mmHg Oxygen saturation 96% on room air 1500 Temperature 37.2 (98.9 F) Heart rate 96/min Respiratory rate 20/min Blood pressure 100/70 mmHg Oxygen saturation 97% on room air Nurses' Notes 1500 Client admitted from the ED for dehydration. Client alert and oriented to person, pla

The client is at risk for developing confusion due to sodium level Upon analyzing cues, the nurse should identify that the client is at risk for confusion due to a sodium level that is greater than the expected reference range. Hypernatremia places the client at risk for a decreased level of consciousness, falls, and seizure activity. Therefore, the nurse should monitor the client's level of consciousness and place the client on fall and seizure precautions.

A nurse is caring for a client who is on the spinal cord injury (SCI) unit Nurses' Notes Day 3, 1700 Client admitted to SCI unit 3 days ago following C7 injury. Skin is cool, pale, and dry to touch. Respirations easy and unlabored. Lung sounds diminished in lower lobes. Abdomen soft and nondistended with active bowel sounds. Client passed a small amount of hard formed stool this AM. Indwelling urinary catheter draining clear yellow urine. Deep tendon reflexes (DTR) are biceps 1+, triceps 1+, pa

The client is most likely experiencing manifestations of pneumonia and autonomic dysreflexia. The nurse should analyze cues from the client's manifestations and determine that the client is most likely experiencing manifestations of pneumonia and autonomic dysreflexia. A client who has a cervical SCI is at risk for respiratory complications because spinal innervation to the respiratory muscles is disrupted. Adventitious breath sounds in the lower lobes bilaterally and a decrease in oxygen saturation to less than 92% can indicate pneumonia. The client's sudden increase in blood pressure, bradycardia, flushing of the skin above the area of the injury, headache, and blurred vision are manifestations of autonomic dysreflexia, which can be a life-threatening condition.

A nurse is caring for a 1-month-old infant. Nurses' Notes 1500: Infant admitted to the pediatric unit. Parent reports infant has been irritable and has vomited after each feeding within the last 3 days.Infant alert, not crying. S1 and S2 noted without murmurs. Lungs clear to auscultation anterior/posterior. Respirations even, unlabored. Abdomen firm. Bowel sounds hypoactive in all 4 quadrants. Small 1 x 1 cm2 mass palpated near umbilicus. Skin warm and dry, turgor with tenting. 1600: Called to

The infant is at highest risk for developing dehydration, as evidenced by the infant's vomiting. When prioritizing hypotheses and using the urgent vs. nonurgent priority framework, the nurse should identify that the infant is at the greatest risk for developing dehydration due to a loss of gastric content from vomiting. An infant with pyloric stenosis presents with projectile vomiting after feeding, distended abdomen, and olive-shaped mass in the epigastrium.

A charge nurse is observing a newly licensed nurse performing a physical assessment on a client. Which of the following actions by the nurse indicates that the charge nurse should intervene? A. While performing a breast examination, the newly licensed nurse discusses techniques of breast self-examination with the client. B. The newly licensed nurse writes detailed notes while performing the head-to-toe assessment. C. The newly licensed nurse uses a penlight to assess for changes in the contou

The newly licensed nurse writes detailed notes while performing the head-to-toe assessment. The newly licensed nurse should record brief notes during the assessment to avoid delays and write more detailed notes after completing the assessment. Incorrect Discussing self-examination techniques with the client while performing the breast examination provides the opportunity to demonstrate correct technique. The newly licensed nurse should use a penlight to provide adequate lighting when assessing contours. The newly licensed nurse should use the dorsal surface of the hand to assess skin temperature because it is sensitive to temperature changes.

A nurse in a provider's office is caring for a client. Nurses' Notes Day 1, 0900: Client is 65-year-old who reports pain and burning on urination. Client states, "I am having trouble making it to the bathroom on time and I'm up throughout the night needing to urinate." Client alert and oriented to person, place, and time. Bilateral breath sounds clear. Respirations even and unlabored. S3 auscultated. Lower extremity edema +1. Radial and pedal pulses +2. Bowel sounds normoactive. Client reports

The nurse is assessing the client. Which of the following assessment findings should the nurse report to the provider? - Urgency - Frequency - Dysuria When recognizing cues, the nurse should identify that the client's report of frequency, dysuria, and urgency are manifestations of a UTI and should be reported to the provider. These manifestations occur due to bacteria invading the urinary tract through the urethra.

A nurse in a provider's office is caring for a client. Nurses' Notes Day 1, 0900: Client is 65-year-old who reports pain and burning on urination. Client states, "I am having trouble making it to the bathroom on time and I'm up throughout the night needing to urinate." Client alert and oriented to person, place, and time. Bilateral breath sounds clear. Respirations even and unlabored. S​3 auscultated. Lower extremity edema +1. Radial and pedal pulses +2. Bowel sounds normoactive. Client repor

The nurse is planning to teach the client how to prevent further UTIs from occurring. Which of the following instructions should the nurse plan to include? Gently cleanse the perineum before intercourse When generating solutions, the nurse should educate the client on how to prevent future UTIs by cleansing the perineum prior to intercourse. During intercourse, bacteria from the skin can enter the urinary tract, causing infection. Incorrect The nurse should instruct the client to void every 2 to 3 hr, rather than every 4 to 6 hr, during the day. Frequent voiding prevents overdistension of the bladder and helps to flush bacteria from the urinary tract. The nurse should instruct the client to drink as much as 2 to 3 L, rather than 4 L, of fluids daily, unless contraindicated by other health conditions. Increased fluids promote renal blood flow and help to flush bacteria from the urinary tract. The nurse should instruct the client to avoid citrus juices, which can irritate the urinary

A nurse is caring for a client who has a new diagnosis of anorexia nervosa. Vital Signs Day 1, 2005: Temperature 35.3° C (95.5° F)Heart rate 60/minRespiratory rate 23/minBlood pressure 90/55 mm HgOxygen saturation 98% on room air Day 2, 0800: Temperature 36.1° C (97° F)Heart rate 65/minRespiratory rate 20/minBlood pressure 88/57 mm HgOxygen saturation 98% on room air Graphic Record Day 1, 2005: Weight 37.5 kg (82.7 lb) Height 162.56 cm (64 in)BMI 14.2 Day 2, 0800: Weight 37.4 kg (82.5 lb

The nurse should first address the client's electrolyte imbalance, followed by the client's fear of weight gain. When analyzing cues, the nurse should first address the client's electrolyte imbalance. The client has hypokalemia, which increases the risk for cardiac arrhythmias. Once the client's medical concerns are addressed, the nurse should then focus on the underlying psychological issues behind the eating disorder, such as the client's fear of weight gain.

A nurse is caring for a client who is pregnant in the acute care setting Nurses' Notes 1400 Client reports a constant low dull backache and painless abdominal tightening for the past 3 hr. Denies any changes in vaginal discharge. External fetal monitor applied. 1430 Contraction pattern: contractions every 4 to 5 min, lasting 30 to 45 seconds, palpate mild in intensity Fetal heart rate: 150/min to 155/min, moderate variability, adequate accelerations present, no decelerations noted. Provider in

The nurse should first address the client's respiratory rate, followed by the client's level of consciousness When prioritizing hypotheses, the nurse should recognize that magnesium sulfate is a central nervous system depressant that can affect respirations, consciousness, and reflexes when toxic blood levels occur. Using the airway, breathing, circulation priority framework, the nurse should plan to first take action to support respirations, followed by action to increase the client's level of consciousness. The nurse should plan to discontinue the magnesium sulfate infusion and administer calcium gluconate as an antidote.

A nurse is caring for a client who has a prescription for a continuous passive motion (CPM) machine following a total knee arthroplasty. Which of the following actions should the nurse take? A. Instruct the client how to adjust the CPM settings for comfort. B. Maintain the client's affected hip in an externally rotated position. C. Turn off the CPM machine during mealtime. D. Store the CPM machine under the client's bed when not in use.

Turn off the CPM machine during mealtime. The nurse should turn off the CPM machine during meals to promote client comfort and dietary intake. Incorrect The provider or the physical therapist prescribes the proper settings for the CPM machine, so the client should not adjust the settings. The nurse should maintain the affected extremity in neutral alignment. The nurse should not store the client's CPM machine under the bed or on the floor when it is not in use because this places the client at increased risk for a wound infection.

A nurse is reviewing the ABG results of a client who has COPD. The results include a pH of 7.3 (7.35 to 7.45), PaO2 56 mm Hg (80 to 100 mmHg), PaCO2 54 mmHg (35 to 45 mmHg), HCO3 26 mEq/L (21 to 28 mEq/L), and SaO2 87%. Which of the following is the correct interpretation of these values? A. Uncompensated metabolic acidosis B. Uncompensated respiratory acidosis C. Compensated respiratory acidosis D. Compensated metabolic acidosis

Uncompensated respiratory acidosis A pH of 7.3 is below the expected reference range and indicates the client has acidosis. A PaCO2 of 54 mm Hg is above the expected reference range, which indicates the acidosis has a respiratory origin when combined with the low pH. The HCO3- of 26 mEq/L is within the expected reference range, indicating that the acidosis is not metabolic in origin and the body has not yet corrected the imbalance through compensation.

A nurse is assessing a client who has skeletal traction for a femur fracture. Which of the following findings should the nurse identify as the priority? A. Muscle spasms of the affected extremity B. A pain rating of 6 on a scale from 0 to 10 C. Upper chest petechiae D. Ecchymosis over the fractured area

Upper chest petechiae The greatest risk to this client is organ damage from fat embolism syndrome, a life-threatening complication of fractures. In fat embolism syndrome, a fat embolus enters the blood stream and can obstruct blood vessels of a major organ, such as the lung, kidney, or brain. Manifestations include petechiae on the upper torso, dyspnea, hypoxia, headache, lethargy, and confusion. Therefore, the nurse should identify this as the priority finding. Incorrect The nurse should reposition the client or check the weights to relieve the client's muscle spasms. However, another finding is the priority. The nurse should provide analgesia to relieve the client's moderate pain level. However, another finding is the priority. The nurse should identify ecchymosis over the fractured area as an expected finding due to localized trauma and provide comfort measures. However, another finding is the priority.

A nurse in an emergency department is caring for a client who is at 9 weeks of gestation and reports nausea and vomiting for the past 2 days. Which of the following findings should the nurse expect? A. Hgb 15 g/dL (12 to 18 g/dL) B. Urine specific gravity 1.052 (1.005 to 1.03) C. Urine osmolality 300 mOsm/kg (50 to 1200 mOsm/kg) D. Hct 44% (37% to 52%)

Urine specific gravity 1.052 (1.005 to 1.03) The nurse should recognize the client's urine specific gravity is significantly elevated above the expected reference range of 1.005 to 1.03. An increased urine specific gravity indicates dehydration from vomiting.

A nurse is caring for a newborn immediately after delivery. Which of the following interventions should the nurse implement to prevent heat loss by conduction? A. Dry the newborn immediately after birth. B. Maintain an ambient room temperature of 24° C (75.2° F). C. Use a protective cover on the scale when weighing the infant. D. Place the newborn's bassinet away from outside windows.

Use a protective cover on the scale when weighing the infant. Heat loss by conduction is a loss of heat between the newborn's skin and the cooler surfaces beneath it. Using a protective cover prevents contact with the scale, which prevents the loss of heat through conduction. Incorrect Drying the newborn prevents heat loss via evaporation, which occurs when a liquid converts to a vapor. This strategy prevents heat loss via convection, which is the flow of heat from the body surface to cooler, ambient air. Moving the bassinet away from outside windows prevents heat loss via radiation. Radiation is the loss of heat from the body surface to cooler, solid surfaces not in direct contact with the newborn, but in relative proximity.

A nurse is providing teaching for a client who has a fracture of the right fibula with a short-leg cast in place and a new prescription for crutches. The client is non-weight-bearing for 6 weeks. Which of the following instructions should the nurse include in the teaching? A. Adjust the crutches for comfort as needed. B. Use a three-point gait. C. Wear leather-soled shoes. D. Advance the affected leg first when walking upstairs.

Use a three-point gait. A three-point crutch gait allows the client to be mobile without bearing weight on the affected extremity. Incorrect A health care provider should initially measure crutches for the client. Clients should not perform their own adjustments to the crutches. Use of improperly fitting crutches increases the risk of injury from falls. The client should wear rubber-soled shoes when using crutches. The client who wears leather-soled shoes has an increased risk of slipping and falling. When walking up stairs, the client should advance the unaffected leg first. When walking down stairs, the client should advance the crutches and the affected leg first and then follow with the unaffected leg.

A nurse is preparing to administer an IM injection to a client who is obese. Which of the following actions should the nurse plan to take? A. Select a 1-inch needle. B. Use a 45º angle when inserting the needle. C. Use the ventrogluteal site. D. Pinch the skin up during injection.

Use the ventrogluteal site. The nurse should use the ventrogluteal site because it has a thick area of muscle and contains no large nerves or blood vessels. Incorrect A 1-inch needle is too short to reach the muscle in a client who has obesity. The nurse should use a 90º angle when inserting the needle into the client's muscle. The nurse should use the Z-track method when administering an IM injection.

A nurse is caring for a client who is postoperative following an appendectomy. Nurses' Notes 1800: Client alert and oriented to person, place, time, and situation.Skin warm and dry.Lungs clear on auscultation Bowel sounds hypoactive in all four quadrants. Urine clear yellow Incisional dressing clean and dry. Client reports pain as 6 on a scale of 0 to 10.1815: Morphine administered as prescribed.2000: Client reports abdominal pain as 10 on a scale of 0 to 10. Client reports nausea, no vomiting.

Which of the following 4 client findings should the nurse report to the provider? - Oxygen saturation - Heart rate - Pain level - Nausea When recognizing cues, the nurse should identify that the findings of pain, nausea, heart rate, and oxygen saturation are unexpected findings for a client who is postoperative following an appendectomy. These findings should be reported to the provider.

The client returns to the provider's office 3 days later. Nurses' Notes Day 1, 0900: Client is 65-year-old who reports pain and burning on urination. Client states, "I am having trouble making it to the bathroom on time and I'm up throughout the night needing to urinate." Client alert and oriented x 3. Bilateral breath sounds clear. Respirations even and unlabored. S​3 auscultated. Lower extremity edema +1. Radial and pedal pulses +2. Bowel sounds normoactive. Client reports no nausea or vomi

Which of the following assessment findings should the nurse report to the provider as unexpected? For each assessment finding, click to specify if the finding is expected or unexpected. Expected findings: urine color, voiding pattern, oxygen saturation, blood pressure Unexpected findings: Temperature, skin, and bowel elimination When taking action, the nurse should identify that the client's urine color, voiding pattern, oxygen saturation, and blood pressure are expected findings and do not need to be reported to the provider. The client's report of orange urine is an expected finding due to the prescribed medication phenazopyridine, which can cause reddish-orange discoloration of urine. The client's voiding pattern is an expected finding due to increased fluid intake of 3 L daily. The client's oxygen saturation is an expected finding due to the client's history of COPD. The client's blood pressure is an expected finding due to the client's history of hypertension. The nurse should

A nurse is caring for an adolescent in the emergency department (ED) Nurses' Notes 0700 Adolescent admitted to ED. Adolescent's parents are concerned about left leg injury that appears to be getting worse. Parents report adolescent has had fever, decreased appetite, and decreased energy within the past 2 days. Adolescent reports leg injury occurred while playing soccer. 0715 Adolescent is alert and oriented to person, place, time, and situation. Adolescent reports left lower leg pain as 4 on

Which of the following findings requires immediate follow up by the nurse? - Skin assessment - Temperature - WBC - Casual blood glucose - Potassium After reviewing the information in the adolescent's EMR and recognizing cues, the nurse should identify that the adolescent has a potential skin infection, such as cellulitis. The skin assessment reveals that the medial lateral aspect of the left leg has a 3 x 3 cm2 area of redness with small pustules, tenderness, and warmth, which can indicate infection. The adolescent's temperature and WBC count are above the expected reference range, which can also indicate infection. The adolescent's casual blood glucose and potassium are above the expected reference range, which can indicate infection or a complication of type 1 diabetes mellitus. The nurse should immediately follow up on these findings because they can indicate infection or other complications.

A nurse is caring for a client in the emergency department (ED). Nurses' Notes 0600: Client admitted to the ED with fatigue, shortness of breath, and weakness for the last 2 days. Client states that they have a history of sickle cell disease (SCD). Client is alert and orientated to person, place, and time. Restless. Client rates generalized pain as a 9 on a scale of 0 to 10. Vital signs taken and blood drawn for laboratory tests. Oxygen 2 L via nasal cannula applied. Awaiting prescription for p

Which of the following interventions should the nurse implement? Select all that apply. - Assess the client's mouth every 8 hr - Assess peripheral circulation hourly - Use humidification with oxygen therapy - Administer IV fluids When taking actions, the nurse should administer IV fluids, use humidification with oxygen therapy, and assess the client's mouth every 8 hr and peripheral circulation hourly. Hydration is a priority when caring for a client in sickle cell crisis because it decreases the rate of cell sickling and can reduce pain. Hypotonic fluids are typically infused at 250 mL/hr for 4 hr. Oxygen administered without humidification can cause drying of the mucous membranes, especially in clients who are already fluid-depleted. Placing humidification on the oxygen therapy promotes comfort and reduces the risk of sores and lesions of the mucous membranes. The nurse should assess the client's peripheral circulation because of the risk of venous occlusion caused by the sickling

A nurse in a provider's office is caring for a client. Nurses' Notes Day 1, 0900: Client is 65-year-old who reports pain and burning on urination. Client states, "I am having trouble making it to the bathroom on time and I'm up throughout the night needing to urinate." Client alert and oriented to person, place, and time. Bilateral breath sounds clear. Respirations even and unlabored. S​3​ auscultated. Lower extremity edema +1. Radial and pedal pulses +2. Bowel sounds normoactive. Client re

Which of the following provider prescriptions should the nurse anticipate for this client? Select the 4 prescriptions the nurse should anticipate. - Collect urine specimen for urinalysis - Educate client on new prescription for sulfamethoxazole/trimethoprim and phenazopyridine - Collect urine specimen for urine culture When analyzing cues, the nurse should anticipate provider prescriptions to collect urine specimens for urinalysis and urine culture and to educate the client on new prescriptions for sulfamethoxazole/trimethoprim and phenazopyridine. The nurse should identify that the client is most likely experiencing a urinary tract infection (UTI). UTIs are diagnosed through urinalysis and urine culture. Clients experiencing a UTI should be prescribed an antibiotic and an analgesic for urinary pain relief and frequency.

A nurse is creating a plan of care for a child who has acute lymphoid leukemia and an absolute neutrophil count of 400/mm3 (2500 to 8000/mm3). Which of the following interventions should the nurse include in the plan? A. Encourage friends and family to visit the child. B. Withhold administering the varicella vaccine to the child. C. Collect a daily urine specimen from the child to check for proteinuria. D. Provide a low-protein diet for the child.

Withhold administering the varicella vaccine to the child. A child who has severe immunodeficiency should not receive a live vaccine due to the risk of developing the disease. Inactivated vaccines can be administered to children who are immunosuppressed. Incorrect The nurse should advise limited contact with friends or family as well as the avoidance of crowded areas to decrease the risk of spreading infection to a child who is immunosuppressed. A child who has severe immunodeficiency should be monitored for the presence of hematuria, not proteinuria, as an adverse effect of chemotherapy. A child who has severe immunodeficiency should eat a high-calorie, high-protein diet to provide adequate nutrients to rebuild their WBC count and fight infection.


Kaugnay na mga set ng pag-aaral

Chapter 55- Ecosystems and Restoration Ecology

View Set

CHEMLEC: Atomic Symbol, Atomic Mass and Isotopes

View Set

Chapter 17: Industrial Supremacy

View Set